6
$\begingroup$

Was wondering if my solution is mathematically accurate enough:

The question in the book yields:

Derive $$ 1=\int_{-\infty}^{\infty} \delta(x-x_i)\ dx_i $$ From $$ f(x)=\int_{-\infty}^{\infty} f(x_i)\delta(x-x_i)\ dx_i $$ [Hint: let $f(x)=1$]

My method is:

$$ f(x)=\int_{-\infty}^\infty f(x_i)\delta(x-x_i)\ dx_i $$

$$ f(x)=1 $$

so $$ 1=\int_{-\infty}^{\infty} \delta(x-x_i)\ dx_i $$

I think this is not correct. Does someone know is this is correct, or how to do it better?

Cheers

$\endgroup$
7
  • 4
    $\begingroup$ Besides the typos mentioned by Babak, the method is correct. There is nothing special there, just applying the definition of Dirak function directly $\endgroup$
    – SBF
    Jan 12, 2013 at 13:05
  • $\begingroup$ It seems fishy. defining f(x)=1 does that also mean f(x_i)=1? $\endgroup$ Jan 12, 2013 at 13:10
  • 1
    $\begingroup$ $f\equiv 1$ is meant $\endgroup$
    – SBF
    Jan 12, 2013 at 13:15
  • 2
    $\begingroup$ For the sake of future readers: the Delta distribution only applies to compactly supported functions, and $f(x)=1$ is not, hence the proof sugested in this question is incorrect. $\endgroup$
    – Did
    Jul 31, 2017 at 12:33
  • $\begingroup$ Regarding the comment above and the answer below about test-functions having to be compactly supported see e.g. Dirac delta and non-test functions and Dirac delta distribution; what can be a test function?. $\endgroup$
    – Winther
    Jul 31, 2017 at 13:20

6 Answers 6

4
$\begingroup$

The error is that in the definition mentioned, f(x) has to be compactly supported and making it constant equal 1 is not correct.

$\endgroup$
1
$\begingroup$

I think you can look at it as a definition.

You can also look at it as the limit of any function that is symmetric around the y axis, which integral is 1, and you make it narrower and narrower, forcing its peak to go to infinity. This function can be a gaussian, like shown in the wikipedia article gif image, or a uniform function, as shown in this Khan Academy video.

$\endgroup$
1
$\begingroup$

https://en.wikipedia.org/wiki/Dirac_delta_function

assumming the following limit as an accepted definition for delta $$ {\displaystyle \delta (x)=\lim _{b\to 0}{\frac {1}{|b|{\sqrt {\pi }}}}e^{-(x/b)^{2}}} $$ by letting $$ {T(x) = {\frac {1}{|b|{\sqrt {\pi }}}}e^{-(x/b)^{2}}} $$ we can $$ \int\limits_{-\infty}^{\infty} T(x)\ \mathrm dx\ = 1 $$ then we can take the limit as b -> 0 and as we can see the the value if the integration is independant of the limit so we can so we get delta in the side of the integration and one on the left side

$\endgroup$
0
$\begingroup$

It looks fine. We know that $$\delta_a(t-t_0)=\frac{1}{2a},\;\; \text{when}\;\;|t-t_0|<a\;\; \text{and}\;\;\delta_a(t-t_0)=0,\;\; \text{when}\;\;|t-t_0|\geq a$$ Here when $a$ tends to zero, the resulted expression which is not a function at all, as you noted above, is: $$\lim\delta_a(t-t_0)=\delta(t-t_0)$$ Using $$\int_{-\infty}^{\infty} \delta_a(t-t_0)\ dt=1$$ we can characterized two peraperties below for that: $$\delta (t-t_0)=\infty,\;\; \text{when}\;\;t=t_0\;\; \text{and}\;\;\delta (t-t_0)=0,\;\; \text{when}\;\;t\neq t_0 $$ and $$\int_{-\infty}^{\infty} \delta(t-t_0)\ dt=1$$

$\endgroup$
6
  • 3
    $\begingroup$ $\delta$ is not a function. In particular, "$\delta(t-t_0) = \infty$ when $t=t_0$" is a meaningless statement. $\endgroup$
    – mrf
    Jan 12, 2013 at 13:19
  • $\begingroup$ @mrf: Maybe saying that $\delta(t-t_0)$ at $t=t_0$ tends to infinity is better? $\endgroup$
    – Mikasa
    Jan 12, 2013 at 13:28
  • 6
    $\begingroup$ Not really. $\delta$ is not a pointwise defined object. It's a distribution and defined in terms of how it acts on test functions. Of course, many elementary presentations cheat and gloss over these things. $\endgroup$
    – mrf
    Jan 12, 2013 at 13:30
  • $\begingroup$ @mrf: Honestly, what I was learned about $\delta$ is that: This relation gets infinity at t_0. Indeed, I frequently draw pictures for what you said about it. Every time I get infinity. Note me kindly my defects and wrong imaginations as your students. Thanks prof. ;-) $\endgroup$
    – Mikasa
    Jan 12, 2013 at 13:36
  • 1
    $\begingroup$ Actually, this is very far from being "fine" since the Delta distribution only applies to compactly supported functions, and $f(x)=1$ is not. $\endgroup$
    – Did
    Jul 31, 2017 at 12:32
0
$\begingroup$

As it was mentioned in other answers and comments, the Dirac- $\delta$ is not a function, and it is not even defined pointwise. It is defined as element of the dual of the space of $C^{\infty}$ functions with compact support.

$$ \langle \delta, \phi\rangle = \phi(0), \quad \phi \in C^{\infty}_c $$

When $f$ is locally integrable, it can be identified with the distribution $$ \langle T_f, \phi\rangle \int_{\mathbb{R}} f(x) \phi(x) \,dx, $$

but this is not the case with the Dirac - $\delta$. Although there are axiomatic definitions of distributions that allow the definition of value of a distribution, and therefore also the concept of integral, this is not very usual. Regarding distributions, what you can easily compute are derivatives: $$ \langle T', \phi\rangle = - \langle T, \phi'\rangle. $$

In particular, this means that any locally integrable function as derivative (in fact, derivatives of any order) in the sense of distributions:

$$ \langle T_f', \phi\rangle = - \int_{\mathbb{R}} f(x) \phi'(x)\, dx. $$

So, although it is sometimes useful to identify the Dirac-$\delta$ with a function andf reason about its integral, this is mostly symbolic and should be taken as a definition, and not something that you can demonstrate using integration/measure.

$\endgroup$
-2
$\begingroup$

If f(x) is a function of x, then it cannot be removed from the integral. Therefore this proof only works if f(x) is a constant.

$\endgroup$

You must log in to answer this question.

Not the answer you're looking for? Browse other questions tagged .